Merge remote branch 'public/master'
[course.git] / latex / problems / Serway_and_Jewett_4 / problem20.69.tex
1 \begin{problem*}{20.69}
2 The $x$ axis is the symmetry axis of a stationary, uniformly charged
3 ring of radius $R$ and charge $Q$ (Fig.~P20.69).  A particle with
4 charge $Q$ and mass $M$ is located at the center of the ring.  When it
5 is displaced slightly, the point charge accelerates along the $x$ axis
6 to infinity.  Show that the ultimate speed of the point charge is
7 \begin{equation}
8  v = \left(\frac{2 k_e Q^2}{MR}\right)^{1/2}
9 \end{equation}
10 \end{problem*} % problem 20.69
11
12 \begin{solution}
13 Conserving energy, the inital energy is entirely electric,
14 \begin{equation}
15  E_i = U_e = k_e \frac{Q^2}{R}
16 \end{equation}
17 because all the ring charge is a distance $R$ from the particle.
18
19 The final energy is entirely kinetic
20 \begin{equation}
21  E_f = K = \frac{1}{2} M v^2
22 \end{equation}
23
24 So
25 \begin{align}
26  k_e \frac{Q^2}{R} = E_i &= E_f = \frac{1}{2} M v^2 \\
27  v &= \ans{\sqrt{\frac{2 k_e Q^2}{M R}}}
28 \end{align}
29 \end{solution}